Logic Games Answers - Ivy Global

6 downloads 1108 Views 776KB Size Report
Preptests 58 Answers and Explanations (By Ivy Global). Ivy Global. Section 4 – Logic Games. Questions 1 – 6. From the rules we know that H must be in 604 or ...
Preptests 58 Answers and Explanations (By Ivy Global)

Section 4 – Logic Games Questions 1 – 6

From the rules we know that H must be in 604 or 605 and M must be in 601, 602, or 603. Because of our H limitation, the G-L-F sequence can’t start in 603—so G has to be in 601 or 602. This in turn limits where L and F can go. There are many limitations in this game, none of which are too concrete, but need to be kept in mind. - We also know that the two monuments in 601 can only be M, S, or G. L and F both come after G, so can’t be in the first slot, and H needs to be in the last two. 1. Type of Question: Global – List This question should be approached with a rule sweep. a) b) c) d) e)

Eliminated by rule #4: must have two monuments in 601 Eliminated by rule #2: H must be in 604 or 605 Eliminated by rule #1: F must come after L Eliminated by rule #3: M must begin before 604 This is the correct answer. It does not violate any rules.

2. Type of Question: Global—Could Be True We know that F has to come after L, so L can’t go in 605. We also know that, because H must be in 604 and 605, we can’t put the L-F sequence in 604 and 605. So, the earliest L can come is in 603. We can put it into a diagram to be sure: M&S G L F H a) b) c) d) e)

L could not be in 601 due to our G-L-F sequence. L could go in 602, but it could also go later. This is the correct answer. L cannot go in 604 due to the L-F block and H requirement. L cannot go in 605 because F must come after it.

3. Type of Question: Global—Fully Determined For this question we’ll need to remember that we have to fully determine, so the answer will tell us exactly where everything goes. The wrong answers will leave ambiguities. a) This forces G and L into 601 and 602 in order to keep our G-L-F sequence intact, and puts M in 601 to follows its rule. But we don’t know where H and S go. Our diagram is incomplete and looks like this: M&G L F H/S H/S b) This forces L into 603 (remember our inference that the latest L can go is in 603), and into 601, but F and H are still up in the air: S&M G L F/H F/H c) This doesn’t tell us anything except that H is in 605. d) We know that G and S will have to be in 601 here, and L will have to be in 603, but the rest aren’t defined yet: G&S M L F/H F/H e) This is the correct answer. Putting S in 604 leaves 605 as the only option for H. Our G-L-F sequence has to fit in 601, 602, and 603. This leaves 601 for M as well: G&M L F S H

Ivy Global

Preptests 58 Answers and Explanations (By Ivy Global) 4. Type of Question: Global—Must Be True In this question we’re looking for the answer that must be true. The wrong answers could be true or false, or must be false. a) This is the correct answer. Since M has to fall in the first three, and F can’t be present until 603 because of the G-L-F sequence, M is always going to be before F. b) This isn’t necessarily true. We saw in 5 a) that F could come before S. c) We`ve seen many times that in the last two slots H and F are interchangeable, so this can`t be right. d) Just like c), we`ve seen H and S interchangeable many times in the last two slots. Eliminate. e) We know that M and G can share the spot in 601—we saw this in 5 a), so this is wrong. 5. Type of Question: Local—Must Be True This is a different kind of question, but it is manageable. What we`re looking for is something that forces L into 602, and if you recognized that this hinges on the early inference we made that L can only fit into 602 or 603. To get it into 602, we simply need to place something else into 603. a) b) c) d) e)

This allows for L to go into either 602 or 603. This allows for L to go into either 602 or 603. This allows for L to go into either 602 or 603. This allows for L to go into either 602 or 603. This is the correct answer. Placing something in 603 forces L into 602: G&M L S H/F H/F

6. Type of Question: Local—Could Be True This new condition has given us a bigger chain than we originally had: G-L-F has become G-LM-F. Since M must be in the first three slots, and since we know that neither H nor F can come first, we know that 601 – 603 must read G&S L M. a) b) c) d) e)

We know that M must be in 603 so this isn’t true. We know that G must be in 601 and L must be in 602 so this can’t work. This is the correct answer. Either F or H could fall into 605. L must be in 602 and M must be in 603, so this isn’t true. S, by virtue of being the only one that could fit in the slow, is in 601. Eliminate.

Questions 7 – 12 This in-and-out game yields a large number of conditional phrases that can be combined together to create one giant chain and one giant contrapositive that looks this: ~L RMT~F ~ V  S ~SV~T~M~RL F This will help us to take a more streamlined approach to the questions. 7. Type of Question: Global—List

Ivy Global

Preptests 58 Answers and Explanations (By Ivy Global) This question should be approached with a rule sweep. a) If F volunteers , then we can follow our chain and see that L must also volunteer. This list is not complete. b) Eliminated based on rule #2: MT. This list is not complete. c) This is the correct answer. It does not break any rules. d) Looking at our chain, we know that R and V cannot volunteer together, so this is inaccurate. e) Based on rule #5, T and V cannot volunteer together. This is inaccurate. 8. Type of Question: Local—Could Be True Looking at our chain’s contrapositive, we know that if V is selected, then T, M, and R are all out, and L is in. F and S are up in the air: IN: V L OUT: T M R ?: F S a) b) c) d) e)

R is out, and so is this answer choice. This is the correct answer. F and S could be either in or out. M can’t volunteer, so this is out. T can’t volunteer, so this is out. T can’t volunteer, so this is out.

9. Type of Question: Local—Cannot Be True/Must Be False Looking at our chain’s contrapositive, the following is known: IN: L OUT: T M R ?: S V F a) b) c) d) e)

We don’t know whether or not F volunteers, so this is out. L has to volunteer, so this must be true. Eliminate. This is the correct answer. R cannot volunteer—this choice CANNOT be true. S can go either way, so this is out. V can go either way, so this is out.

10. Type of Question: Local—Could Be True This triggers our main chain, and yields the following: IN: M T S OUT: F V ?: L R a) b) c) d) e)

F has to be out, so this is not right. This is the correct answer. L could volunteer, or could not, so this could be true. V must be out, so this is not right. S must be in, so this is not right. T must be in, so this is not right.

11. Type of Question: Local—Must Be True This triggers the contrapositive chain and yield the following:

Ivy Global

Preptests 58 Answers and Explanations (By Ivy Global) IN: F L OUT: T M R ?: S V a) b) c) d) e)

This is the correct answer. L must volunteer. S could go either way, so this doesn’t have to be true. V could go either way, so this doesn’t have to be true. S and V could both volunteer, so this doesn’t have to be true. Only one of S and V could volunteer, so again, this doesn’t have to be true.

12. Type of Question: Global—Must Be True This question asks us to use what we know in our chains to find pairs that must be together. To find this, we’ll have to find triggers where when one member of the pair is out, the other must be in. For example, ~SV (contrapositive ~VS), means that we’ll never see a grouping without either S or V. a) We will never see F and T volunteering together, but they could be out together. So this isn’t true. b) This is the correct answer. ~LRM and (contrapostive ~M~RL) fits our prephrase. c) L and V are either both in or both out, so this isn’t right. d) R and S are either both in or both out, so this isn’t right. e) T and S are either both in or both out, so this isn’t right. Questions 13 – 17 ********This question gives us a lot to work with, but it’s pretty manageable if we organize it well. With all our deductions made, the diagram looks like this: __ __ int __ __ __

__ __ Q __ dom dom __ get fly

13. Type of Question: Global—Must Be True This question should be approached with a rule sweep. a) Eliminated based on rules #5 and #6: Since Q is the only Flyhigh domestic flight, it must come last, because int’l flights come before domestic ones, and Getaway domestics come before Flyhighs. b) Also eliminated based on rules #5 and #6: Since Q is the only Flyhigh domestic flight, it must come last, because int’l flights come before domestic ones, and Getaway domestics come before Flyhighs. c) Also eliminated based on rules #5 and #6: Since Q is the only Flyhigh domestic flight, it must come last, because int’l flights come before domestic ones, and Getaway domestics come before Flyhighs. d) This is the correct answer. It does not violate any rules. e) Eliminated based on rule #6: P, an int’l flight, must come before R, a domestic flight.

Ivy Global

Preptests 58 Answers and Explanations (By Ivy Global)

14. Type of Question: Global—Could Be True We know that Q can’t go in any position other than 5, so we can eliminate answer choice e) right away. The other four planes could be placed in the second slot. Let’s look at set-ups to prove this to ourselves: S P T R Q P R S T Q P S R T Q P T R S Q d) This is the correct answer. 15. Type of Question: Local—Must Be False Since P is an international flight, and all international flights come before domestics, we’ll have to make S, which now comes before P, also an international flight. We also know that Q and R are both domestic flights, and have to come after all internationals. This means that the last two flights must be domestic, the first two international, and the third can be either. The S-P block then can come only in spot 1 and 2 or in spots 2 and 3: S P T R Q or T S P R Q a) This is fine. b) This is the correct answer. S cannot fall into 3, because it would push P into 4—and 4 already is filled by R. c) This is fine. d) This is fine. e) This is fine. 16. Type of Question: Global—Must Be True We need to attack the questions. a) This isn’t necessarily true. We just saw in 15 that P could be in 2 or 3. b) This is the correct answer and if we made our deductions properly, we would have found this when we diagrammed. c) This isn’t necessarily true. We just saw in 15 that R can go in 4. d) This isn’t necessarily true. We just saw in 15 that S can go in 2. e) This isn’t necessarily true. T is a bit of a floater and can go in almost any spot. 17. Type of Question: Local—Could Be True Except We need to attack the question. a) b) c) d) e)

This can happen: P R S T Q This can happen: P R S T Q This is the correct answer. Since Q must be last, R and T don’t fit into spots 4 and 5. This can happen: T P S R Q This can happen: T P S R Q

Ivy Global

Preptests 58 Answers and Explanations (By Ivy Global) Questions 18 – 23

This is a very unique set of conditional rules, and when you diagram it out it should reveal a criss-crossed puzzle. There are a few important things to keep in mind: L is a floater and does not yield anything. T results in only one condition. M results in three conditions. H, P S and W result in two each. 18. Type of Question: Global—List This question should be approached using a rule sweep. a) b) c) d) e)

Eliminated by rule #1: if history is taken then statistics cannot be taken Eliminated by rule #2: if music is taken then physics cannot be taken This is the correct answer. It does not violate any rules. Eliminated by rule # 3: if writing is taken then physics cannot be taken Eliminated by rule #2: if music is taken then theatre cannot be taken

19. Type of Question: Global—Could Be True To solve this we need to maximize the number of entities that are in. How? Since L is a floater, let’s put it in. It doesn’t force anything out and it adds one course. We know that M forces 3 courses out, so let’s put M out. It can yield more in than out. T only has one condition and that’s forcing M out. M’s already out, so we can put T in. We’re left with H P S W. We can only have one of H and S, and we can’t pair W with P or with S. So we can have maximum two from this group—H and P/W, or W and H, or P and W. We have L, T, and two from our H P S W group. This gives us 4 courses. d) This is the correct answer. 20. Type of Question: Local—Could Be True This question puts two courses out: P and W. It’s a bit tough because putting courses out doesn’t yield any direct inferences for us, but if we keep our numbers in mind this question gets easier. We know right off the bat that M won’t be selected. It would eliminate H, S, and T, leaving only M and L. We need at least three courses, so we can put M in the ‘out’ pile. We’re left with H L S T, and of this we need to select three. We know that we have to select L, because the combination of H S T violates rule #1. We’re left with this: IN: L H/S T OUT: P W M H/S a) The student must take L, so this is out. b) This is the correct answer. c) We can’t have both H and S out. One is out, and the other is in.

Ivy Global

Preptests 58 Answers and Explanations (By Ivy Global) d) The student must take L, so this is out. e) T has to be in by virtue of the H/S split, so this is out. 21. Type of Question: Local—Must Be True We know from our rules and diagram that if M is selected, then H, P, and T are all out. We’re left with L, S, and W. Since S and W can’t occur together, and we need at least three courses, then our floater, L, must be selected. Our diagram looks like this: IN: M L S/W OUT: H P T S/W a) b) c) d) e)

W could be either in or out. T must be out. S could be either in or out. P must be out. This is the correct answer.

22. Type of Question: Global—Must Be True We’ll have to attack this question using our past work and working through each answer choice. a) We saw in question 21 that both H and S could be out, so this isn’t right. b) This is the correct answer. If we knock out L and T we’ll need three of H M P S W, and each of these knocks out two others, so there’s no way to make the minimum with this set-up. c) We saw in question 18 that both L and W could be out, so this is wrong. d) Question 20 put both M and P out, so this is wrong. e) Question 21 allowed for both T and W to be out, so this doesn’t have to be true. 23. Type of Question: Local—Must Be True (New Rule) Often these questions are the most time-consuming and difficult ones in a logic game, but this one is not too difficult if we just organize our thoughts. We need to replace the rule M~P and ~T. What do we know about M? Well, it can’t be taken with H (rule 1) or P or T (rule 2). This means it can only be taken with L S W. As long as the answer tells us this we’ll have filled in the gap we created. a) b) c) d) e)

This isn’t a “must” condition, and it also doesn’t restrict M from being taken with T. This is the correct answer. This doesn’t help—we need to focus on the M condition. This doesn’t help—we need to focus on the M condition. It’s not that P and T need to be taken together, but that M doesn’t allow for EITHER P or T to be taken.

Ivy Global